Lecture 24: Intro/Framework for Psych Nursing

¡Supera tus tareas y exámenes ahora con Quizwiz!

The nurse is conducting an educational seminar for health care providers on mental health coverage and reimbursement for clients. Which statement is most important to include in the teaching? "Health insurance is employer based, and serious mental illness can lead to job loss." "Health insurance readily pays for physical problems, so always use a physical diagnosis as the primary problem." "Medicare and Medicaid do not cover mental illnesses." "Private insurance always covers mental illnesses."

"Health insurance is employer based, and serious mental illness can lead to job loss." Most health insurance is supplied through employers. Clients with mental disorders often have difficultly holding down employment due to their symptoms. Health care providers must be aware that clients with mental illness are at risk for losing their jobs and health insurance. The client's diagnosis should be consistent with his or her presentation and may not always be physical. Other options such as Medicare and Medicaid may be available in these situations for qualifying clients. Private insurance does not always cover mental illnesses. p. 58

The student nurse is describing characteristics of furniture on the psychiatric unit. Which description is most accurate? "The furniture for inpatient rooms tends to be lightweight, so if it is thrown it will not cause too much injury." "The furniture for inpatient rooms has rounded corners so that head injuries do not occur." "The furniture for inpatient rooms is heavy and durable." "There is no furniture in the inpatient rooms to prevent injuries."

"The furniture for inpatient rooms is heavy and durable." Furniture for inpatient rooms tends to be heavy and durable so that it cannot be thrown or dismantled and used for weapons. Lightweight furniture is not used because it is easier to lift and throw, and it is easy to disassemble and used as weaponry. Furniture edges should be rounded or padded for protection of small children. Seclusion rooms contain no furniture and have padded walls to prevent injury. Inpatient rooms have furniture. p. 56

The nurse is assessing a client for eligibility to receive psychiatric home care. What criteria must the client meet to receive reimbursement for psychiatric home care? Select all that apply. Current psychiatric diagnosis Age 65 years or older Wheelchair-bound status Under the care of a primary care physician (PCP) Homebound status Eligible for Medicare

-Current psychiatric diagnosis -Under the care of a primary care physician (PCP) -Homebound status Criteria for reimbursement of psychiatric home care include a psychiatric diagnosis, being under the care of a PCP, and homebound status. Homebound means the client cannot safely leave home, leaving home causes undue stress, or the nature of the illness results in refusal to leave home. It is not a stipulation for clients to be 65 years or older, wheelchair bound, or eligible for Medicare. Test-Taking Tip: Be alert for details about what you are being asked to do. In this question type, you are asked to select all options that apply to a given situation or patient. All options likely relate to the situation, but only some of the options may relate directly to the situation. p. 54

The nurse working at a Veteran's Administration (VA) clinic assesses a client with posttraumatic stress disorder (PTSD). What is the approximate prevalence of PTSD in veterans of the Iraq and Afghanistan wars? 3% 7% 14% 24%

14% For veterans of the Iraq and Afghanistan wars, PTSD prevalence is about 14%. More veterans than 3% experienced PTSD. The prevalence of PTSD in the general population is approximately 7%. Fewer veterans than 24% experienced PTSD. Test-Taking Tip: The most reliable way to ensure that you select the correct response to a multiple choice question is to recall it. Depend on your learning and memory to furnish the answer to the question. To do this, read the stem, and then stop! Do not look at the response options yet. Try to recall what you know and, based on this, what you would give as the answer. After you have taken a few seconds to do this, then look at all of the choices and select the one that most nearly matches the answer you recalled. It is important that you consider all the choices and not just choose the first option that seems to fit the answer you recall. Remember the distractors. The second choice may look okay, but the fourth choice may be worded in a way that makes it a slightly better choice. If you do not weigh all the choices, you are not maximizing your chances of correctly answering each question. p. 57

A patient diagnosed with major depression has lost 20 pounds in one month, has chronic low self-esteem, and a plan for suicide. The patient has taken an antidepressant medication for 1 week. Which nursing intervention has the highest priority? a. Implement suicide precautions. b. Offer high-calorie snacks and fluids frequently. c. Assist the patient to identify three personal strengths. d. Observe patient for therapeutic effects of antidepressant medication.

ANS: A Implementing suicide precautions is the only option related to patient safety. The other options, related to nutrition, self-esteem, and medication therapy, are important but are not priorities.

When a nurse assesses an older adult patient, answers seem vague or unrelated to the questions. The patient also leans forward and frowns, listening intently to the nurse. An appropriate question for the nurse to ask would be: a. "Are you having difficulty hearing when I speak?" b. "How can I make this assessment interview easier for you?" c. "I notice you are frowning. Are you feeling annoyed with me?" d. "You're having trouble focusing on what I'm saying. What is distracting you?"

ANS: A The patient's behaviors may indicate difficulty hearing. Identifying any physical need the patient may have at the onset of the interview and making accommodations are important considerations. By asking if the patient is annoyed, the nurse is jumping to conclusions. Asking how to make the interview easier for the patient may not elicit a concrete answer. Asking about distractions is a way of asking about auditory hallucinations, which is not appropriate because the nurse has observed that the patient seems to be listening intently.

A patient says, "Please don't share information about me with the other people." How should the nurse respond? a. "I will not share information with your family or friends without your permission, but I share information about you with other staff." b. "A therapeutic relationship is just between the nurse and the patient. It is up to you to tell others what you want them to know." c. "It depends on what you choose to tell me. I will be glad to disclose at the end of each session what I will report to others." d. "I cannot tell anyone about you. It will be as though I am talking about my own problems, and we can help each other by keeping it between us."

ANS: A A patient has the right to know with whom the nurse will share information and that confidentiality will be protected. Although the relationship is primarily between the nurse and patient, other staff needs to know pertinent data. The other incorrect responses promote incomplete disclosure on the part of the patient, require daily renegotiation of an issue that should be resolved as the nurse-patient contract is established, and suggest mutual problem solving. The relationship must be patient centered. See relationship to audience response question.

Which statement shows a nurse has empathy for a patient who made a suicide attempt? a. "You must have been very upset when you tried to hurt yourself." b. "It makes me sad to see you going through such a difficult experience." c. "If you tell me what is troubling you, I can help you solve your problems." d. "Suicide is a drastic solution to a problem that may not be such a serious matter."

ANS: A Empathy permits the nurse to see an event from the patient's perspective, understand the patient's feelings, and communicate this to the patient. The incorrect responses are nurse- centered (focusing on the nurse's feelings rather than the patient's), belittling, and sympathetic.

A nurse explains to the family of a mentally ill patient how a nurse-patient relationship differs from social relationships. Which is the best explanation? a. "The focus is on the patient. Problems are discussed by the nurse and patient, but solutions are implemented by the patient." b. "The focus shifts from nurse to patient as the relationship develops. Advice is given by both, and solutions are implemented." c. "The focus of the relationship is socialization. Mutual needs are met, and feelings are shared openly." d. "The focus is creation of a partnership in which each member is concerned with growth and satisfaction of the other."

ANS: A Only the correct response describes elements of a therapeutic relationship. The remaining responses describe events that occur in social or intimate relationships.

At what point in the nurse-patient relationship should a nurse plan to first address termination? a. During the orientation phase b. At the end of the working phase c. Near the beginning of the termination phase d. When the patient initially brings up the topic

ANS: A The patient has a right to know the conditions of the nurse-patient relationship. If the relationship is to be time-limited, the patient should be informed of the number of sessions. If it is open-ended, the termination date will not be known at the outset, and the patient should know that the issue will be negotiated at a later date. The nurse is responsible for bringing up the topic of termination early in the relationship, usually during the orientation phase.

A nurse asks a patient, "If you had fever and vomiting for 3 days, what would you do?" Which aspect of the mental status examination is the nurse assessing? a. Behavior b. Cognition c. Affect and mood d. Perceptual disturbances

ANS: B Assessing cognition involves determining a patient's judgment and decision making. In this case, the nurse would expect a response of "Call my doctor" if the patient's cognition and judgment are intact. If the patient responds, "I would stop eating" or "I would just wait and see what happened," the nurse would conclude that judgment is impaired. The other options refer to other aspects of the examination.

Before assessing a new patient, a nurse is told by another health care worker, "I know that patient. No matter how hard we work, there isn't much improvement by the time of discharge." The nurse's responsibility is to: a. document the other worker's assessment of the patient. b. assess the patient based on data collected from all sources. c. validate the worker's impression by contacting the patient's significant other. d. discuss the worker's impression with the patient during the assessment interview.

ANS: B Assessment should include data obtained from both the primary and reliable secondary sources. The nurse, bearing in mind the possible effects of counter-transference, should evaluate biased assessments by others as objectively as possible.

A patient presents to the emergency department with mixed psychiatric symptoms. The admission nurse suspects the symptoms may be the result of a medical problem. Lab results show elevated BUN (blood urea nitrogen) and creatinine. What is the nurse's next best action? a. Report the findings to the health care provider. b. Assess the patient for a history of renal problems. c. Assess the patient's family history for cardiac problems. d. Arrange for the patient's hospitalization on the psychiatric unit.

ANS: B Elevated BUN (blood urea nitrogen) and creatinine suggest renal problems. Renal dysfunction can often imitate psychiatric disorders. The nurse should further assess the patient's history for renal problems and then share the findings with the health care provider.

Which entry in the medical record best meets the requirement for problem-oriented charting? a. "A: Pacing and muttering to self. P: Sensory perceptual alteration related to internal auditory stimulation. I: Given fluphenazine HCL (Prolixin) 2.5 mg po at 0900 and went to room to lie down. E: Calmer by 0930. Returned to lounge to watch TV." b. "S: States, 'I feel like I'm ready to blow up.' O: Pacing hall, mumbling to self. A: Auditory hallucinations. P: Offer haloperidol (Haldol) 2 mg po. I: Haloperidol (Haldol) 2 mg po given at 0900. E: Returned to lounge at 0930 and quietly watched TV." c. "Agitated behavior. D: Patient muttering to self as though answering an unseen person. A: Given haloperidol (Haldol) 2 mg po and went to room to lie down. E: Patient calmer. Returned to lounge to watch TV." d. "Pacing hall and muttering to self as though answering an unseen person. haloperidol (Haldol) 2 mg po administered at 0900 with calming effect in 30 minutes. Stated, 'I'm no longer bothered by the voices.'"

ANS: B Problem-oriented documentation uses the first letter of key words to organize data: S for subjective data, O for objective data, A for assessment, P for plan, I for intervention, and E for evaluation. The distracters offer examples of PIE charting, focus documentation, and narrative documentation.

A nurse wants to assess an adult patient's recent memory. Which question would best yield the desired information? a. "Where did you go to elementary school?" b. "What did you have for breakfast this morning?" c. "Can you name the current president of the United States?" d. "A few minutes ago, I told you my name. Can you remember it?"

ANS: B The patient's recall of a meal provides evidence of recent memory. Two incorrect responses are useful to assess immediate and remote memory. The other distracter assesses the patient's fund of knowledge.

After formulating the nursing diagnoses for a new patient, what is a nurse's next action? a. Designing interventions to include in the plan of care b. Determining the goals and outcome criteria c. Implementing the nursing plan of care d. Completing the spiritual assessment

ANS: B The third step of the nursing process is planning and outcome identification. Outcomes cannot be determined until the nursing assessment is complete and nursing diagnoses have been formulated.

A nurse assesses a confused older adult. The nurse experiences sadness and reflects, "The patient is like one of my grandparents...so helpless." Which response is the nurse demonstrating? a. Transference c. Catastrophic reaction b. Countertransference d. Defensive coping reaction

ANS: B Countertransference is the nurse's transference or response to a patient that is based on the nurse's unconscious needs, conflicts, problems, or view of the world. See relationship to audience response question.

A nurse wants to demonstrate genuineness with a patient diagnosed with schizophrenia. The nurse should: a. restate what the patient says. b. use congruent communication strategies. c. use self-revelation in patient interactions. d. consistently interpret the patient's behaviors.

ANS: B Genuineness is a desirable characteristic involving awareness of one's own feelings as they arise and the ability to communicate them when appropriate. The incorrect options are undesirable in a therapeutic relationship.

Which descriptors exemplify consistency regarding nurse-patient relationships? Select all that apply. a. Encouraging a patient to share initial impressions of staff b. Having the same nurse care for a patient on a daily basis c. Providing a schedule of daily activities to a patient d. Setting a time for regular sessions with a patient e. Offering solutions to a patient's problems

ANS: B, C, D Consistency implies predictability. Having the same nurse see the patient daily and provide a daily schedule of patient activities and a set time for regular sessions will help a patient predict what will happen during each day and develop a greater degree of security and comfort. Encouraging a patient to share initial impressions of staff and giving advice are not related to consistency and would not be considered a therapeutic intervention.

A patient is very suspicious and states, "The FBI has me under surveillance." Which strategies should a nurse use when gathering initial assessment data about this patient? Select all that apply. a. Tell the patient that medication will help this type of thinking. b. Ask the patient, "Tell me about the problem as you see it." c. Seek information about when the problem began. d. Tell the patient, "Your ideas are not realistic." e. Reassure the patient, "You are safe here."

ANS: B, C, E During the assessment interview, the nurse should listen attentively and accept the patient's statements in a nonjudgmental way. Because the patient is suspicious and fearful, reassuring safety may be helpful, although trust is unlikely so early in the relationship. Saying that medication will help or telling the patient that the ideas are not realistic will undermine development of trust between the nurse and patient.

1. A nurse assessed a patient who reluctantly participated in activities, answered questions with minimal responses, and rarely made eye contact. What information should be included when documenting the assessment? Select all that apply. a. The patient was uncooperative b. The patient's subjective responses c. Only data obtained from the patient's verbal responses d. A description of the patient's behavior during the interview e. Analysis of why the patient was unresponsive during the interview

ANS: B, D Both content and process of the interview should be documented. Providing only the patient's verbal responses would create a skewed picture of the patient. Writing that the patient was uncooperative is subjectively worded. An objective description of patient behavior would be preferable. Analysis of the reasons for the patient's behavior would be speculation, which is inappropriate.

What information is conveyed by nursing diagnoses? Select all that apply. a. Medical judgments about the disorder b. Unmet patient needs currently present c. Goals and outcomes for the plan of care d. Supporting data that validate the diagnoses e. Probable causes that will be targets for nursing interventions

ANS: B, D, E Nursing diagnoses focus on phenomena of concern to nurses rather than on medical diagnoses.

A novice nurse tells a mentor, "I want to convey to my patients that I am interested in them and that I want to listen to what they have to say." Which behaviors will be helpful in meeting the nurse's goal? Select all that apply. a. Sitting behind a desk, facing the patient b. Introducing self to a patient and identifying own role c. Maintaining control of discussions by asking direct questions d. Using facial expressions to convey interest and encouragement e. Assuming an open body posture and sometimes mirror imaging

ANS: B, D, E Trust is fostered when the nurse gives an introduction and identifies his or her role. Facial expressions that convey interest and encouragement support the nurse's verbal statements to that effect and strengthen the message. An open body posture conveys openness to listening to what the patient has to say. Mirror imaging enhances patient comfort. A desk would place a physical barrier between the nurse and patient. A face-to-face stance should be avoided when possible and a less intense 90- or 120-degree angle used to permit either party to look away without discomfort.

An adolescent asks a nurse conducting an assessment interview, "Why should I tell you anything? You'll just tell my parents whatever you find out." Which response by the nurse is appropriate? a. "That isn't true. What you tell us is private and held in strict confidence. Your parents have no right to know." b. "Yes, your parents may find out what you say, but it is important that they know about your problems." c. "What you say about feelings is private, but some things, like suicidal thinking, must be reported to the treatment team." d. "It sounds as though you are not really ready to work on your problems and make changes."

ANS: C Adolescents are very concerned with confidentiality. The patient has a right to know that most information will be held in confidence but that certain material must be reported or shared with the treatment team, such as threats of suicide, homicide, use of illegal drugs, or issues of abuse. The incorrect responses are not true, will not inspire the confidence of the patient, or are confrontational.

A patient begins a new program to assist with building social skills. In which part of the plan of care should a nurse record the item, "Encourage patient to attend one psychoeducational group daily"? a. Assessment b. Analysis c. Implementation d. Evaluation

ANS: C Interventions are the nursing prescriptions to achieve the outcomes. Interventions should be specific.

Select the most appropriate label to complete this nursing diagnosis: ___________ related to feelings of shyness and poorly developed social skills as evidenced by watching television alone at home every evening. a. Deficient knowledge b. Ineffective coping c. Social isolation d. Powerlessness

ANS: C Nursing diagnoses are selected based on the etiological factors and assessment findings, or evidence. In this instance, the evidence shows social isolation that is caused by shyness and poorly developed social skills.

A newly admitted patient diagnosed with major depression has gained 20 pounds over a few months and has suicidal ideation. The patient has taken an antidepressant medication for 1 week without remission of symptoms. Select the priority nursing diagnosis. a. Imbalanced nutrition: more than body requirements b. Chronic low self-esteem c. Risk for suicide d. Hopelessness

ANS: C Risk for suicide is the priority diagnosis when the patient has both suicidal ideation and a plan to carry out the suicidal intent. Imbalanced nutrition, hopelessness, and chronic low self-esteem may be applicable nursing diagnoses, but these problems do not affect patient safety as urgently as would a suicide attempt.

Which behavior shows that a nurse values autonomy? The nurse: a. suggests one-on-one supervision for a patient who has suicidal thoughts. b. informs a patient that the spouse will not be in during visiting hours. c. discusses options and helps the patient weigh the consequences. d. sets limits on a patient's romantic overtures toward the nurse.

ANS: C A high level of valuing is acting on one's belief. Autonomy is supported when the nurse helps a patient weigh alternatives and their consequences before the patient makes a decision. Autonomy or self-determination is not the issue in any of the other behaviors.

What is the desirable outcome for the orientation stage of a nurse-patient relationship? The patient will demonstrate behaviors that indicate: a. self-responsibility and autonomy. c. rapport and trust with the nurse. b. a greater sense of independence. d. resolved transference.

ANS: C Development of rapport and trust is necessary before the relationship can progress to the working phase. Behaviors indicating a greater sense of independence, self-responsibility, and resolved transference occur in the working phase.

During which phase of the nurse-patient relationship can the nurse anticipate that identified patient issues will be explored and resolved? a. Preorientation c. Working b. Orientation d. Termination

ANS: C During the working phase, the nurse strives to assist the patient in making connections among dysfunctional behaviors, thinking, and emotions and offers support while alternative coping behaviors are tried.

After several therapeutic encounters with a patient who recently attempted suicide, which occurrence should cause the nurse to consider the possibility of countertransference? a. The patient's reactions toward the nurse seem realistic and appropriate. b. The patient states, "Talking to you feels like talking to my parents." c. The nurse feels unusually happy when the patient's mood begins to lift. d. The nurse develops a trusting relationship with the patient.

ANS: C Strong positive or negative reactions toward a patient or over-identification with the patient indicate possible countertransference. Nurses must carefully monitor their own feelings and reactions to detect countertransference and then seek supervision. Realistic and appropriate reactions from a patient toward a nurse are desirable. One incorrect response suggests transference. A trusting relationship with the patient is desirable. See relationship to audience response question.

Termination of a therapeutic nurse-patient relationship has been successful when the nurse: a. avoids upsetting the patient by shifting focus to other patients before the discharge. b. gives the patient a personal telephone number and permission to call after discharge. c. discusses with the patient changes that happened during the relationship and evaluates outcomes. d. offers to meet the patient for coffee and conversation three times a week after discharge.

ANS: C Summarizing and evaluating progress help validate the experience for the patient and the nurse and facilitate closure. Termination must be discussed; avoiding discussion by spending little time with the patient promotes feelings of abandonment. Successful termination requires that the relationship be brought to closure without the possibility of dependency-producing ongoing contact.

The desired outcome for a patient experiencing insomnia is, "Patient will sleep for a minimum of 5 hours nightly within 7 days." At the end of 7 days, review of sleep data shows the patient sleeps an average of 4 hours nightly and takes a 2-hour afternoon nap. The nurse will document the outcome as: a. consistently demonstrated. b. often demonstrated. c. sometimes demonstrated. d. never demonstrated.

ANS: D Although the patient is sleeping 6 hours daily, the total is not one uninterrupted session at night. Therefore, the outcome must be evaluated as never demonstrated. See relationship to audience response question.

Nursing behaviors associated with the implementation phase of nursing process are concerned with: a. participating in mutual identification of patient outcomes. b. gathering accurate and sufficient patient-centered data. c. comparing patient responses and expected outcomes. d. carrying out interventions and coordinating care.

ANS: D Nursing behaviors relating to implementation include using available resources, performing interventions, finding alternatives when necessary, and coordinating care with other team members.

The desired outcome for a patient experiencing insomnia is, "Patient will sleep for a minimum of 5 hours nightly within 7 days." At the end of 7 days, review of sleep data shows the patient sleeps an average of 4 hours nightly and takes a 2-hour afternoon nap. What is the nurse's next action? a. Continue the current plan without changes. b. Remove this nursing diagnosis from the plan of care. c. Write a new nursing diagnosis that better reflects the problem. d. Examine interventions for possible revision of the target date.

ANS: D Sleeping a total of 5 hours at night remains a reasonable outcome. Extending the period for attaining the outcome may be appropriate. Examining interventions might result in planning an activity during the afternoon rather than permitting a nap. Continuing the current plan without changes is inappropriate. Removing this nursing diagnosis from the plan of care would be correct when the outcome was met and the problem resolved. Writing a new nursing diagnosis is inappropriate because no other nursing diagnosis relates to the problem.

A nurse documents: "Patient is mute despite repeated efforts to elicit speech. Makes no eye contact. Inattentive to staff. Gazes off to the side or looks upward rather than at speaker." Which nursing diagnosis should be considered? a. Defensive coping b. Decisional conflict c. Risk for other-directed violence d. Impaired verbal communication

ANS: D The defining characteristics are more related to the nursing diagnosis of impaired verbal communication than to the other nursing diagnoses.

A nurse prepares to assess a new patient who moved to the United States from Central America three years ago. After introductions, what is the nurse's next comment? a. "How did you get to the United States?" b. "Would you like for a family member to help you talk with me?" c. "An interpreter is available. Would you like for me to make a request for these services?" d. "Are you comfortable conversing in English, or would you prefer to have a translator present?"

ANS: D The nurse should determine whether a translator is needed by first assessing the patient for language barriers. Accuracy of the assessment depends on the ability to communicate in a language that is familiar to the patient. Family members are not always reliable translators. An interpreter may change the patient's responses; a translator is a better resource.

A patient states, "I'm not worth anything. I have negative thoughts about myself. I feel anxious and shaky all the time. Sometimes I feel so sad that I want to go to sleep and never wake up." Which nursing intervention should have the highest priority? a. Self-esteem-building activities b. Anxiety self-control measures c. Sleep enhancement activitie d. Suicide precautions

ANS: D The nurse would place a priority on monitoring and reinforcing suicide self-restraint because it relates directly and immediately to patient safety. Patient safety is always a priority concern. The nurse should monitor and reinforce all patient attempts to control anxiety, improve sleep patterns, and develop self-esteem, while giving priority attention to suicide self-restraint.

Select the best outcome for a patient with the nursing diagnosis: Impaired social interaction related to sociocultural dissonance as evidenced by stating, "Although I'd like to, I don't join in because I don't speak the language very well." Patient will: a. show improved use of language. b. demonstrate improved social skills. c. become more independent in decision making. d. select and participate in one group activity per day.

ANS: D The outcome describes social involvement on the part of the patient. Neither cooperation nor independence has been an issue. The patient has already expressed a desire to interact with others. Outcomes must be measurable. Two of the distracters are not measurable.

Which statement made by a patient during an initial assessment interview should serve as the priority focus for the plan of care? a. "I can always trust my family." b. "It seems like I always have bad luck." c. "You never know who will turn against you." d. "I hear evil voices that tell me to do bad things."

ANS: D The statement regarding evil voices tells the nurse that the patient is experiencing auditory hallucinations and may create risks for violence. The other statements are vague and do not clearly identify the patient's chief symptom.

At what point in an assessment interview would a nurse ask, "How does your faith help you in stressful situations?" During the assessment of: a. childhood growth and development b. substance use and abuse c. educational background d. coping strategies

ANS: D When discussing coping strategies, the nurse might ask what the patient does when upset, what usually relieves stress, and to whom the patient goes to talk about problems. The question regarding whether the patient's faith helps deal with stress fits well here. It would be out of place if introduced during exploration of the other topics.

A nurse assesses an older adult patient brought to the emergency department by a family member. The patient was wandering outside saying, "I can't find my way home." The patient is confused and unable to answer questions. Select the nurse's best action. a. Record the patient's answers to questions on the nursing assessment form. b. Ask an advanced practice nurse to perform the assessment interview. c. Call for a mental health advocate to maintain the patient's rights. d. Obtain important information from the family member.

ANS: D When the patient (primary source) is unable to provide information, secondary sources should be used, in this case, the family member. Later, more data may be obtained from other information sources familiar with the patient. An advanced practice nurse is not needed for this assessment; it is within the scope of practice of the staff nurse. Calling a mental health advocate is unnecessary. See relationship to audience response question.

Which remark by a patient indicates passage from orientation to the working phase of a nurse-patient relationship? a. "I don't have any problems." b. "It is so difficult for me to talk about problems." c. "I don't know how it will help to talk to you about my problems." d. "I want to find a way to deal with my anger without becoming violent."

ANS: D Thinking about a more constructive approach to dealing with anger indicates a readiness to make a behavioral change. Behavioral change is associated with the working phase of the relationship. Denial is often seen in the orientation phase. It is common early in the relationship, before rapport and trust are firmly established, for a patient to express difficulty in talking about problems. Stating skepticism about the effectiveness of the nurse-patient relationship is more typically a reaction during the orientation phase.

A nurse is talking with a patient, and 5 minutes remain in the session. The patient has been silent most of the session. Another patient comes to the door of the room, interrupts, and says to the nurse, "I really need to talk to you." The nurse should: a. invite the interrupting patient to join in the session with the current patient. b. say to the interrupting patient, "I am not available to talk with you at the present time." c. end the unproductive session with the current patient and spend time with the interrupting patient. d. tell the interrupting patient, "This session is 5 more minutes; then I will talk with you."

ANS: D When a specific duration for sessions has been set, the nurse must adhere to the schedule. Leaving the first patient would be equivalent to abandonment and would destroy any trust the patient had in the nurse. Adhering to the contract demonstrates that the nurse can be trusted and that the patient and the sessions are important. The incorrect responses preserve the nurse-patient relationship with the silent patient but may seem abrupt to the interrupting patient, abandon the silent patient, or fail to observe the contract with the silent patient.

The nurse records this entry in a patient's progress notes: Patient escorted to unit by ER nurse at 2130. Patient's clothing was dirty. In interview room, patient sat with hands over face, sobbing softly. Did not acknowledge nurse or reply to questions. After several minutes, abruptly arose, ran to window, and pounded. Shouted repeatedly, "Let me out of here." Verbal intervention unsuccessful. Order for stat dose 2 mg haloperidol PO obtained; medication administered at 2150. By 2215, patient stopped shouting and returned to sit wordlessly in chair. Patient placed on one-to-one observation. How should this documentation be evaluated? a. Uses unapproved abbreviations b. Contains subjective material c. Too brief to be of value d. Excessively wordy e. Meets standards

ANS: E This narrative note describes patient appearance, behavior, and conversation. It mentions that less-restrictive measures were attempted before administering medication and documents patient response to medication. This note would probably meet standards. A complete nursing assessment would be in order as soon as the patient is able to participate. Subjective material is absent from the note. Abbreviations are acceptable.

A psychiatric client is participating in a partial hospitalization program (PHP). What is a typical client schedule in the PHP? Half day Monday through Friday About 6 hours Monday through Friday Half day each on Saturday and Sunday About 3 to 4 hours each on Saturday and Sunday

About 6 hours Monday through Friday The typical client schedule for PHP is Monday through Friday for about 6 hours, because they are "partially hospitalized." Half day Monday through Friday is a typical schedule for intensive outpatient programs (IOPs). Programs are usually done Monday through Friday and exclude weekend days. p. 54

What is the inability to recognize the need for care due to disorganized thinking called? Stigma Denial Anosognosia Abstinence

Anosognosia Anosognosia means the inability to recognize the need for care due to disorganized thinking. Stigma is a negative perception of individuals with mental illness. Denial is the lack of acceptance or willingness to understand or deal with a problem. Abstinence is refraining from something. Test-Taking Tip: Multiple choice questions can be challenging, because students think they will recognize the right answer when they see it or that the right answer will somehow stand out from the other choices. This is a dangerous misconception. The more carefully the question is constructed, the more each of the choices will seem like the correct response. p. 52

A mentally ill gunman opens fire in a crowded movie theater, killing six people and injuring others. Which comment about this event by a member of the community most clearly shows the stigma of mental illness? A) "Gun control laws are inadequate in our country" B) "It's frightening to feel that it is not safe to go to a movie theater" C) "All these people with mental illness are violent and should be locked up" D) "These events happen because American families no longer go to church together"

C) "All these people with mental illness are violent and should be locked up"

In what is the client participating when attending group therapy to enhance coping skills? Cognitive behavior therapy Occupational therapy Physical therapy Recreational therapy

Cognitive behavior therapy Coping skills are taught and enhanced by participating in cognitive behavioral groups that focus on symptom management. Occupational therapy provides an opportunity to practice life skills that may have been delayed or altered. Physical therapy focuses on physical conditioning and mobility. Recreational therapy activities are used to improve emotional, physical, cognitive, and social well-being. p. 56

What responsibilities does the psychiatric mental health registered nurse carry out? Develop, implement, and evaluate plans of care Maintain oversight of restraint and seclusion Coordinate care by the treatment team Gather data and identify the psychiatric diagnosis Prescribe psychotropic medications for clients Monitor behavior, affect, and mood

Develop, implement, and evaluate plans of care Maintain oversight of restraint and seclusion Coordinate care by the treatment team Monitor behavior, affect, and mood The responsibilities of the psychiatric mental health registered nurse include developing, implementing, and evaluating plans of care; maintaining oversight of restraint use and seclusion; coordinating care provided for the client by the treatment team; and monitoring behavior, affect, and mood. Psychiatric mental health registered nurses do not diagnose the mental health condition but do identify the nursing diagnosis. Psychiatric mental health registered nurses do not prescribe psychotropic drugs; they do monitor and report drug effectiveness to health care providers. p. 57

A client experiencing an exacerbation of schizophrenia with psychotic features is reluctant to seek help despite family encouragement. What is the most common reason a client may not seek help for a mental illness? Lack of financial stability results in refusal of care by a health care provider. Disorganized thoughts impede the ability to recognize the need for care. Lack of family support results in the client's need for independence. The client is afraid of accumulating a large hospital bill.

Disorganized thoughts impede the ability to recognize the need for care. A client with a mental illness such as schizophrenia with psychotic features often experiences disorganized thinking that impedes the ability to recognize the need for care. Lack of financial stability is unlikely to cause a health care provider not to treat the client. Nonprofit hospitals accept any client regardless of the ability to pay. There is no indication that the client has lack of family support or is fearful of owing money for health care. p. 52

The nurse is determining discharge living arrangements for a hospitalized mental health client. Which behavior demonstrated by the client would fulfill requirements for financial reimbursement for placement in a psychiatric home environment? Expresses paranoia regarding police Engages in numerous compulsive rituals Experiences both auditory and visual hallucinations Experiences panic attacks when among strangers outside of the home

Experiences panic attacks when among strangers outside of the home To qualify for reimbursement, clients must have a psychiatric diagnosis, be under the care of a primary care practitioner, and be homebound. Because the client experiences panic attacks outside of the home, he or she may be homebound and therefore qualify for financial reimbursement. Paranoia regarding police, compulsive rituals, and auditory and visual hallucinations do not necessarily qualify a client for financial reimbursement in a psychiatric home environment. p. 54

The nurse is writing discharge instructions of a mental health client. What information should discharge instructions include? Select all that apply. Follow-up appointments Medication instructions Education and prescriptions Instructions on arranging temporary housing Physician's home phone number

Follow-up appointments Medication instructions Education and prescriptions Discharge instructions for clients with mental illness should include follow-up appointments, medication instructions, and education and prescriptions. The social worker should make housing arrangements for the client prior to discharge. The nurse should never give clients the physician's home phone number without permission from the physician. p. 56

As Election Day nears, a mental health nurse studies the position statements of various candidates for federal offices. Which candidate's plan would the nurse interpret as most supportive of services for clients diagnosed with mental illness? Full-parity insurance coverage for mental illness Coverage for biologically based mental illnesses Reimbursement for initial treatment of addictions Managed care oversight for mental illness services

Full-parity insurance coverage for mental illness Full-parity insurance coverage ensures payment for mental health disorders equal to that of physical disorders; this is most supportive of services for clients diagnosed with mental illness. Coverage for biologically based mental illnesses is less supportive because it excludes mental health disorders that may not have a biological basis. Reimbursement for initial treatment of addictions limits coverage to a specific type of mental health disorder and for a limited amount of time. Managed care oversight for mental illness services does not ensure parity of coverage. p. 58

What is the most common method of inpatient suicide? Drowning Self-inflicted gunshot Hanging Cutting wrists

Hanging The most common method of inpatient suicide is hanging. Drowning is unlikely in the health care setting. Clients are searched for weapons such as knives or firearms upon entry into the organization, so suicide by gunshot or cutting wrists is unlikely. p. 56

When discussing the trend of treating mental health clients in community care environments, the nurse identifies which treatment-related event as the trigger for the shift away from traditional hospitalization? Increase in available psychopharmacological agents Increase in voluntary commitments to traditional hospital settings Increased diagnosis of clients with serious and persistent mental illnesses Increase in Medicare and Medicaid coverage for clients in psychiatric hospitals

Increase in available psychopharmacological agents The treatment-related event that triggered the shift away from traditional hospitalization was the increased availability of psychopharmacological agents. Gradually, more psychopharmacological agents were added to treat psychosis, depression, anxiety, and other disorders. The treatment of mental illness expanded from specialists in psychiatry to general practitioners. Increased voluntary commitment to traditional hospital settings did not occur. There was no increase in clients being diagnosed with mental illness. Medicare and Medicaid did not provide coverage for clients in psychiatric hospitals during this time. Test-Taking Tip: Read the question carefully before looking at the answers: (1) Determine what the question is really asking, and look for key words; (2) read each answer thoroughly, and see if it completely covers the material the question asks; and (3) narrow the choices by immediately eliminating answers you know are incorrect. p. 53

Which client care service provides structure in which clients eat meals, receive medication, attend activities, and participate in individual and group therapies on a schedule? Psychiatric rehabilitation services Psychiatric home care Intensive outpatient programs Inpatient care

Inpatient care Inpatient care provides structure, in which clients eat meals, receive medications (if necessary), attend activities, and participate in individual and group therapies on a schedule. Psychiatric rehabilitation services focus on the end goal of integration into society and include coordinated care from a variety of disciplines. Psychiatric home care can be provided by any medical health professional, but typically by nurses with previous inpatient care experience. Intensive outpatient programs function as intermediate programs between inpatient and outpatient care by primary health care providers, nurses, and social workers. Test-Taking Tip: Start by reading each of the answer options carefully. Usually at least one of them will be clearly wrong. Eliminate this one from consideration. Now you have reduced the number of response choices by one and improved the odds. Continue to analyze the options. If you can eliminate one more choice in a four-option question, you have reduced the odds to 50/50. While you are eliminating the wrong choices, recall often occurs. One of the options may serve as a trigger that causes you to remember what a few seconds ago had seemed completely forgotten. p. 56

A client presents to the clinic and states, "I've been on heroin for the past 5 years. It has destroyed my life. I want to quit but I get very sick when I go cold turkey. Can you help me?" Which plan of care would most meet this client's needs? 12-step program at the local church Drug counseling and education Inpatient care for detoxification Self-help group

Inpatient care for detoxification To begin recovery from heroin addition, the client needs to be admitted to a detoxification unit to stabilize physical symptoms during withdrawal. The 12-step program is an outpatient group therapy approach that is helpful in maintaining sobriety. Drug counseling and education should be done once the client undergoes detoxification. Self-help groups are not appropriate for the client who needs to stop taking heroin in a controlled and safe environment. Test-Taking Tip: Key words or phrases in the question stem such as first, primary, early, or best are important. Similarly, words such as only, always, never, and all in the alternatives are frequently evidence of a wrong response. No real absolutes exist in life; however, every rule has its exceptions, so answer with care. p. 58

A pediatric psychiatric registered nurse is explaining treatment options to the parents of a 12-year-old child. Which options are available for pediatric psychiatric care? Inpatient pediatric psychiatric care Outpatient pediatric psychiatric care Pediatric residential treatment centers Geriatric psychiatric care Veteran's administration centers Forensic psychiatric care

Inpatient pediatric psychiatric care Outpatient pediatric psychiatric care Pediatric residential treatment centers A child in need of psychiatric care has the same options as adults; however, they take place in a pediatric setting. Pediatric psychiatric care includes inpatient services, outpatient services, and residential treatment centers. Geriatric centers are specialized for the care of elderly psychiatric clients. Veteran's administration centers manage clients who are mentally affected by their military experiences. Forensic psychiatric care takes place in the prison system. p. 57

The nurse educator is teaching a group of student nurses about the 1999 Olmstead decision. What was the main outcome of the Olmstead decision? It declared psychiatric clients must be deemed incompetent to receive care. It suggested that psychiatric clients be mandated into residential treatment centers. It deemed that mental institutions were safe and effective treatment options. It described mental illness as a disability and institutionalization as a violation of the client's rights.

It described mental illness as a disability and institutionalization as a violation of the client's rights. In the 1999 Olmstead decision, the Supreme Court ruled that institutionalizing clients with mental illness was in violation of the Americans with Disabilities Act. Institutionalization was described as "unjustified isolation." Clients who are declared incompetent receive care despite objections. The Olmstead decision did not mandate clients into residential treatment centers or describe mental institutions as safe and effective treatment options. Test-Taking Tip: Multiple choice questions can be challenging, because students think they will recognize the right answer when they see it or that the right answer will somehow stand out from the other choices. This is a dangerous misconception. The more carefully the question is constructed, the more each of the choices will seem like the correct response. p. 53

The community health nurse plays an important role in prevention and management of mental illness. What service is the community health nurse most likely to perform? Conducting data analysis on individual clients for diagnosis of mental illness Leading an educational session for a community group on stress-reduction techniques Obtaining and recording vital signs on a group of school-aged children with attention-deficit/hyperactivity disorder (ADHD) Perform lice screenings for school-age children within the community

Leading an educational session for a community group on stress-reduction techniques The community health nurse's role in mental health care is focused on prevention and management of mental illness. The community is the nurse's client. As such, leading an educational session on stress-reduction techniques for a community group is an example of community nursing care to prevent or manage stress. Conducting data analysis on individual clients for mental illness is the role of a mental health registered nurse. Obtaining and recording vital signs on a group of school-aged children with ADHD is the role of a school nurse or pediatric nurse specialist. Performing lice screenings is done within school systems and at individual physicians' offices. p. 54

Identify safety considerations for inpatient room design on a psychiatric care unit. Strong, sturdy closet rods and hooks Hand-held sprinkler showerheads Locked windows Removal of shoestrings Platform beds

Locked windows Removal of shoestrings Platform beds Safety considerations for inpatient psychiatric rooms include locked windows to deter jumping, removal of shoestrings and other devices that could be used for hanging, and platform beds to prevent possible crushing. Closet rods and hooks, towel bars, and shower rods are constructed to break if subjected to more than minimal weight to prevent hanging. Flush-mounted sprinkler and showerheads are used rather than hand-held sprinklers that could possibly be fashioned into a hanging device. Test-Taking Tip: After you have eliminated one or more choices, you may discover that two of the options are very similar. This can be helpful, because it may mean that one of these look-alike answers is the best choice and the other is a very good distractor. Test both of these options against the stem. Ask yourself which one completes the incomplete statement grammatically and which one answers the question more fully and completely. The option that best completes or answers the stem is the one you should choose. Here, too, pause for a few seconds, give your brain time to reflect, and recall may occur. pp. 54, 56

The nurse admits a client experiencing hallucinations and delusional thinking to an inpatient mental health unit. The plan of care will require which service to occur first? Social history Psychiatric history Medical assessment Psychological evaluation

Medical assessment Medical assessment is the first step of inpatient care to first rule out comorbid conditions. Social history and psychiatric history should be taken after the client has been deemed physiologically stable. The appropriate member of the treatment team should conduct a full psychological evaluation after admission. p. 56

What is the difference between intensive outpatient programs (IOPs) and partial hospitalization programs (PHPs)? Only IOPs function as intermediate steps between inpatient and outpatient care. More time is spent with clients in PHPs than in IOPs. Only PHPs are located within hospitals. Clients are more closely monitored for relapse in IOPs than in PHPs.

More time is spent with clients in PHPs than in IOPs. The difference between IOPs and PHPs is the amount of time spent with the clients. Both groups tend to be Monday through Friday. IOPs are usually half a day, while PHPs are longer (about 6 hours per day). Both function as intermediate steps between inpatient and outpatient care. Both are usually located within general hospitals, psychiatric hospitals, or in community settings. Clients are closely monitored for relapse in both programs. p. 54

What does the concept of "least restrictive environment" mean? Mobility is discouraged, and the client is restrained. Restraints are only used with the client's permission. Necessary care is provided while permitting personal freedom. Necessary care is provided in the outpatient setting only.

Necessary care is provided while permitting personal freedom. The "least restrictive environment" means the client receives necessary care while being allowed the greatest personal freedom. Mobility is not discouraged; rather, the focus and intent are on client safety. As such, a client may be restrained for safety and continually assessed for readiness to remove them. Client permission is not required for restraint use. The "least restrictive environment" addresses clients in an inpatient setting, such as a hospital, or an outpatient setting, such as a nursing home. 52

An insulin-dependent client with diabetes presents to the emergency room confused, incoherent, and hallucinating. What is the priority assessment for this client? Determine the client's mental health history. Place the client in physical restraints for safety. Obtain a finger stick glucose reading. Consult the on-call psychiatrist.

Obtain a finger stick glucose reading. The priority assessment in this situation is to assess the client's glucose level. The client's signs and symptoms could be due to hypoglycemia. Physical problems are to be ruled out before mental health issues are addressed. The client's mental health history is not a priority. A client should never be placed in restraints unless there is threat of harm to the self or others and lesser methods have not been effective. The psychiatrist should not be consulted until a physical problem is ruled out. p. 56

One treatment goal for a hospitalized client is to be discharged to a residential treatment environment. Which priority intervention will the nurse include in the client's plan of care? Art therapy to reduce the effects of the illness Recreational therapy to improve social well-being Physical therapy to address any existing musculoskeletal disabilities Occupational therapy to assist in assuming skills needed to regain independence

Occupational therapy to assist in assuming skills needed to regain independence Residential treatment programs are structured short- or long-term 24-hour living environments in which individuals are provided with varying levels of supervision and support. Because hospitalization may lessen a client's independence, occupational therapy to improve the client's ability to perform activities of daily living will be a priority in this client's plan of care. Art therapy, recreational therapy, and physical therapy may be necessary as well but these would be of lower priority than occupational therapy to reestablish independence. Test-Taking Tip: You have at least a 25% chance of selecting the correct response in multiple choice items. If you are uncertain about a question, eliminate the choices you believe are wrong, and then call on your knowledge, skills, and abilities to choose from the remaining responses. p. 56

A client is visiting the primary health care provider (PCP). The client complains of difficulty sleeping. He states, "Those snakes on the floor scare me. There are so many! My daughter says there are no snakes, but she is trying to get my money." What are the disadvantages of mental illness being treated by PCPs? PCP appointment time constraints Complicated assessments when psychiatric disorders accompany physical complaints Limited PCP training in psychiatry Greater comfort in being treated in a familiar setting PCPs lacking expertise in diagnosis and treatment of psychiatric disorders Minimal stigma attached to receiving psychiatric care from PCPs

PCP appointment time constraints Limited PCP training in psychiatry PCPs lacking expertise in diagnosis and treatment of psychiatric disorders Disadvantages of being treated for mental illness by the PCP include appointment time constraints, limited training in psychiatry, and lack of expertise in the diagnosis and treatment of psychiatric disorders. Appointments with the PCP are usually scheduled in 15-minute increments, not permitting enough time for a detailed psychiatric assessment. Psychiatric disorders are often accompanied by physical complaints, but the PCP may overemphasize the physical over the psychiatric. PCPs may have limited training in psychiatry and lack expertise required to diagnose and manage these disorders. Benefits of PCP care are that the client may feel greater comfort in receiving treatment in a familiar setting, and the degree of stigma attached to mental illness might be minimized. p. 53

In order to reduce fragmented care and improve services, patient-centered medical homes (PCMHs) were developed and received strong support from the Affordable Care Act of 2010. What are the key characteristics of PCMHs? Select all that apply. Patient-centered care Comprehensive care Duplicated services Coordination of care Improved access to care Continuous evidence-based care

Patient-centered care Comprehensive care Coordination of care Improved access to care Continuous evidence-based care PCMHs have five key characteristics: 1) patient-centered care, 2) comprehensive care, 3) coordination of care, 4) improved access to care and needed services, and 5) a systems approach making use of evidence-based practice, provided in a continuous feedback loop of evaluation and quality improvement. PCMHs are intended to eliminate fragmented or duplicated care by using a patient-centered approach providing access to physical health, behavioral health, and supportive community and social services. p. 53

What individuals with mental illness are paid or volunteer to use their experiences to provide recovery-oriented services and support others with mental illness? Recreation therapists Art therapists Peer specialists Licensed professional counselors

Peer specialists Peer specialists are individuals with serious mental illness who receive training to use their experience to provide recovery-oriented services and support others with mental illness. Recreation therapists are typically bachelor's prepared and may be licensed by the state or be nationally certified. Recreational therapists provide activities used to improve emotional, physical, cognitive, and social well-being. Art therapists are prepared at the master's level in art therapy and registered through a professional organization. Licensed professional counselors possess a master's degree in psychology, counseling, or a related field and are licensed by the state. They assess and diagnose psychiatric conditions and provide individual, family, and group counseling. p. 55

What services may be available as part of residential treatment programs? Psychoeducation Physical therapy Vocational training Activities of daily living training Recreational therapy

Psychoeducation Vocational training Activities of daily living training Clients in residential treatment programs may receive psychoeducation about the disease process and medications, vocational training, and training for activities of daily living. Physical therapy and recreational therapy are not considered part of the residential treatment program. p. 56

The client with a recent onset of bipolar disorder is admitted to the mental health unit. The nurse is most likely to arrange which type of group therapy for the client? Occupational therapy Skilled nursing care Psychoeducational group therapy Cognitive behavioral therapy

Psychoeducational group therapy Psychoeducational group therapy focuses on specific psychiatric disorders, medication, goal setting, life planning, and recovery, which would greatly improve the newly diagnosed client's quality of life. Occupational therapy gives clients an opportunity to practice life skills that have been delayed, hampered, or eroded, which is not indicated for this client. Skilled nursing care is appropriate for clients who require continued therapy after discharge from acute care. Cognitive behavioral therapy groups focus on coping skills. p. 56

What does psychiatric rehabilitation emphasize? Recovery and integration into society Reduction in physiological symptoms Need for aggressive interventions Reintegration of multiple personalities into one

Recovery and integration into society Psychiatric rehabilitation is a social model that emphasizes and supports recovery and integration into society. Reduction of psychological symptoms may be an aspect of individual care but is not the emphasis of this model. Aggressive interventions are not provided in the psychiatric rehabilitation model. Reintegration of multiple personalities into one is not the emphasis of psychiatric rehabilitation. p. 54

A client diagnosed with major depressive disorder tells the community mental health nurse, "I usually spend all day watching television. If there's nothing good to watch, I just sleep or think about my problems." What is the nurse's most appropriate action? Suggest that the client instead call some friends. Refer the client for counseling with a recreation therapist. Refer the client for counseling with an occupational therapist. Tell the client that watching television and thinking about problems worsens depression.

Refer the client for counseling with a recreation therapist. A recreation therapist can help the client find activities to do during free time that may better improve emotional, physical, cognitive, and social well-being. Suggesting that the client call friends could make the client feel worse if this is not possible given the client's support system or level of motivation for social engagement. Occupational therapists work with clients to develop the practical and necessary skills of daily independent living. Advising the client that watching television and thinking about problems will only make depression worse conveys judgment without helping the client find better health-promoting activities. p. 55

A client with newly diagnosed bipolar disorder is in need of minimal supervision and support, psychoeducation for symptom management, and understanding of lithium therapy. What is the best care setting to meet this client's needs? Intensive inpatient hospitalization Residential treatment center Intensive outpatient program General medical floor

Residential treatment center This client is best suited to a residential treatment center. The residential treatment centers are short- or long-term 24-hour living environments that provide varying levels of supervision and support, individualized psychoeducation, and access to community support services to achieve maximal independence. The intensive inpatient hospitalization is for clients with suicidal ideation, aggressive impulses, or other issues that require crisis stabilization and behavior modification. Because this client is stable and needs minimal supervision, the intensive outpatient program is unnecessary. A general medical floor is not equipped to fully address the client's mental health needs. p. 56

Following the death of his spouse, an elderly client presents to the emergency room with chest pain, sweaty palms, and dizziness. What is the priority of care in this situation? Administer an antianxiety medication Obtain an order for an antidepressant medication Rule out possible cardiac disease Ask the client if he has a suicide plan

Rule out possible cardiac disease The priority of care in this situation is to rule out cardiac disease. Once the client is medically cleared, depression and anxiety should be addressed. Administering an antianxiety medication may be beneficial but is not the priority in this situation. There is not enough information to indicate the need for an antidepressant medication. The client should be assessed for suicide risk after he is medically cleared. p. 52

Identify the goals of inpatient psychiatric care. Reduction of hallucinations Prevention of delusions Regulation of repetitive behaviors Safety Stabilization Crisis intervention

Safety Stabilization Crisis intervention The goals of inpatient psychiatric care include client safety, stabilization, and crisis intervention. Reduction in hallucinations may be an individual client goal. Prevention of delusions, although desirable, may not be a realistic goal for some clients. Regulation of repetitive behaviors may be an individual goal. pp. 54, 56

The quality of care provided by state hospitals has improved dramatically. What is the clinical role of state hospitals? Provide services for homeless clients Limit exposing the community to the client Extend services once provided in early mental institutions Serve the most seriously ill clients

Serve the most seriously ill clients The clinical role of state hospitals is to serve the most seriously ill clients. Providing services for homeless clients is not the focus or intent of state hospitals. Limiting exposure to the community is not consistent with providing high quality health services, and it violates the Americans with Disabilities Act. Services provided at present day psychiatric hospitals are not intended to become an extension of the early institutions. p. 56

The mental health nurse is working with another team member to find temporary housing for a psychiatric client. Which health care team member is most suited to assist in locating appropriate temporary housing? Social worker Licensed professional counselor Occupational therapist Psychiatric mental health advanced practice nurse

Social worker The team member best suited to assist the nurse with locating appropriate temporary housing is the social worker. Social workers may provide counseling and plan for supportive services such as housing, health care, and treatment after the client returns to the community. Licensed professional counselors assess and diagnose psychiatric conditions and provide individual, family, and group counseling. Occupational therapists assist clients to develop or regain independent living skills, activities of daily living, and role performance that have been affected by mental disorders. The psychiatric mental health advanced practice nurse provides assessment, diagnosis, and treatment in outpatient settings. p. 55

The nurse educator is teaching a group of nursing students about the president's vision for mental health care in America. The nurse educator discusses the President's New Freedom Commission on Mental Health. What was the purpose of this group? Studying the relationship between mental illness and physical symptoms Studying the mental health system and making recommendations for its transformation Examining the number of people with mental illness in the United States Collecting data on the prevalence of mental illness in homeless Americans

Studying the mental health system and making recommendations for its transformation The President's New Freedom Commission on Mental Health was charged with studying the mental health system and making recommendations for its transformation. The groups did not study the relationship between mental illness and physical symptoms, examine the number of people with mental illness, or collect data on the number of homeless people with mental illness. pp. 58-59

Which factors may influence the level of psychiatric care given to clients in the current health care system? Substance abuse problems Familial support Medical problems Financial stability Acceptance of treatment Potential for relapse

Substance abuse problems Medical problems Acceptance of treatment Potential for relapse Decisions for the level of care given to psychiatric clients may be influenced by such factors as concurrent psychiatric or substance abuse problems, medical problems, acceptance of treatment, social supports, and disease chronicity or potential for relapse. Familial support consideration, although helpful, may be an unnecessary consideration depending on other available support services and client acceptance. Financial stability is not a consideration when determining appropriate and needed care. p. 53

A community mental health nurse is preparing to address a national parent-teacher organization. Which statement concerning the availability of residential treatment centers for emotionally disturbed children in the United States should the nurse include? Centers have increased but only marginally since 2004. The numbers of centers grew substantially between 2004 and 2008. Availability of these centers has declined steadily over the last decade. Availability of residential treatment beds has remained steady over the last decade.

The numbers of centers grew substantially between 2004 and 2008. The number of residential treatment centers for emotionally disturbed children increased from 33,835 to 50,063 with available beds per 100,000 civilian population increasing from 11.4 to 16.5 for the years between 2004 and 2008. This increase is significant, not marginal. Availability of centers has not declined, nor has availability of treatment beds remained stagnant; both increased as the quantity of centers increased. p. 55

What is a common misconception about psychiatric nurses in acute care? They have just as much education as registered nurses on other care units. They just talk to clients and lose their nursing skills. They are very good at monitoring effects of psychotropic drugs. They may be uncomfortable inserting an intravenous catheter (IV) or administering IV medications.

They just talk to clients and lose their nursing skills. A common misconception about psychiatric nurses is that they "just talk" to clients. Therapeutic communication is a nursing skill that takes time to master. Psychiatric nurses are experts in therapeutic communication. Psychiatric nurses have just as much education as registered nurses in other types of health care. Psychiatric nurses are experienced at monitoring intended psychotropic drug effectiveness. Psychiatric nurses may be uncomfortable performing skills that are not common in the mental healthcare setting such as inserting IV catheters or administering IV medications. p. 57

A patient is hospitalized for depression and suicidal ideation after their spouse asks for a divorce. Select the nurse's most caring comment. a. "Let's discuss some means of coping other than suicide when you have these feelings." b. "I understand why you're so depressed. When I got divorced, I was devastated too." c. "You should forget about your marriage and move on with your life." d. "How did you get so depressed that hospitalization was necessary?"

a. "Let's discuss some means of coping other than suicide when you have these feelings."

A student nurse tells the instructor, "I don't need to interact with my patients. I learn what I need to know by observation." The instructor can best interpret the nursing implications of Sullivan's theory to the student by responding: a. "Nurses cannot be isolated. We must interact to provide patients with opportunities to practice interpersonal skills." b. "Observing patient interactions can help you formulate priority nursing diagnoses and appropriate interventions." c. "I wonder how accurate your assessment of the patient's needs can be if you do not interact with the patient." d. "Noting patient behavioral changes is important because these signify changes in personality."

a. "Nurses cannot be isolated. We must interact to provide patients with opportunities to practice interpersonal skills."

Carolina is surprised when her patient does not show for a regularly scheduled appointment. When contacted, the patient states, "I don't need to come see you anymore. I have found a therapy app on my phone that I love." How should Carolina respond to this news? a. "That sounds exciting, would you be willing to visit and show me the app?" b. "At this time, there is no real evidence that the app can replace our therapy." c. "I am not sure that is a good idea right now, we are so close to progress." d. "Why would you think that is a better option than meeting with me?"

a. "That sounds exciting, would you be willing to visit and show me the app?"

A patient states, "I'm starting cognitive behavioral therapy. What can I expect from the sessions?" Which responses by the nurse are appropriate? Select all that apply. a. "The therapist will be active and questioning." b. "You may be given homework assignments." c. "The therapist will ask you to describe your dreams." d. "The therapist will help you look at ideas and beliefs you have about yourself." e. "The goal is to increase your subjectivity about thoughts that govern your behavior."

a. "The therapist will be active and questioning." b. "You may be given homework assignments." d. "The therapist will help you look at ideas and beliefs you have about yourself."

A 26-month-old child displays negative behaviors. The parent says, "My child refuses toilet training and shouts, 'No!' when given direction. What do you think is wrong?" Select the nurse's best reply. a. "This is normal for your child's age. The child is striving for independence." b. "The child needs firmer control. Punish the child for disobedience and say, 'No.'" c. "There may be developmental problems. Most children are toilet trained by age 2 years." d. "Some undesirable attitudes are developing. A child psychologist can help you develop a remedial plan."

a. "This is normal for your child's age. The child is striving for independence."

The family of a patient whose insurance will not pay for continuing hospitalization considers transferring the patient to a public psychiatric hospital. The family expresses concern that the patient will "never get any treatment." Which reply by the nurse would be most helpful? a. "Under the law, treatment must be provided. Hospitalization without treatment violates patients' rights." b. "That's a justifiable concern because the right to treatment extends only to the provision of food, shelter, and safety." c. "Much will depend on other patients, because the right to treatment for a psychotic patient takes precedence over the right to treatment of a patient who is stable." d. "All patients in public hospitals have the right to choose both a primary therapist and a primary nurse."

a. "Under the law, treatment must be provided. Hospitalization without treatment violates patients' rights."

A bill introduced in Congress would reduce funding for the care of people diagnosed with mental illnesses. A group of nurses write letters to their elected representatives in opposition to the legislation. Which role have the nurses fulfilled? a. Advocacy b. Attending c. Recovery d. Evidence-based practice

a. Advocacy

During an admission assessment and interview, which channels of information communication should the nurse be monitoring? Select all that apply. a. Auditory b. Visual c. Written d. Tactile e. Olfactory

a. Auditory b. Visual d. Tactile e. Olfactory

When should a nurse be most alert to the possibility of communication errors resulting in harm to the patient? a. Change of shift report b. Admission interviews c. One-to-one conversations with patients d. Conversations with patient families

a. Change of shift report

A patient who immigrated to the United States from Honduras was diagnosed with schizophrenia. The patient took an antipsychotic medication for 3 weeks but showed no improvement. Which resource should the treatment team consult for information on more effective medications for this patient? a. Clinical algorithm b. Clinical pathway c. Clinical practice guideline d. International Statistical Classification of Diseases and Related Health Problems (ICD)

a. Clinical algorithm

Which action by a psychiatric nurse best supports a patient's right to be treated with dignity and respect? a. Consistently addressing a patient by title and surname. b. Strongly encouraging a patient to participate in the unit milieu. c. Discussing a patient's condition with another health care provider in the elevator. d. Informing a treatment team that a patient is too drowsy to participate in care planning.

a. Consistently addressing a patient by title and surname.

A nurse says, "When I was in school, I learned to call upset patients by name to get their attention; however, I read a descriptive research study that says that this approach does not work. I plan to stop calling patients by name." Which statement is the best appraisal of this nurse's comment? a. One descriptive research study rarely provides enough evidence to change practice. b. Staff nurses apply new research findings only with the help from clinical nurse specialists. c. New research findings should be incorporated into clinical algorithms before using them in practice. d. The nurse misinterpreted the results of the study. Classic tenets of practice do not change.

a. One descriptive research study rarely provides enough evidence to change practice.

A nurse assesses that a patient is suspicious and frequently manipulates others. Using the Freudian theory, these traits are related to which psychosexual stage? a. Oral b. Anal c. Phallic d. Genital

a. Oral

What principle about nurse-patient communication should guide a nurse's fear about "saying the wrong thing" to a patient? a. Patients tend to appreciate a well-meaning person who conveys genuine acceptance, respect, and concern for their situation. b. The patient is more interested in talking to you than listening to what you have to say and so is not likely to be offended. c. Considering the patient's history, there is little chance that the comment will do any actual harm. d. Most people with a mentally illness have by necessity developed a high tolerance of forgiveness.

a. Patients tend to appreciate a well-meaning person who conveys genuine acceptance, respect, and concern for their situation.

A nurse at the mental health center prepares to administer a scheduled injection of haloperidol decanoate (Haldol depot) to a patient with schizophrenia. As the nurse swabs the site, the patient shouts, "Stop! I don't want to take that medicine anymore. I hate the side effects." Select the nurse's best initial action. a. Stop the medication administration procedure and say to the patient, "Tell me more about the side effects you've been having." b. Say to the patient, "Since I've already drawn the medication in the syringe, I'm required to give it, but let's talk to the doctor about skipping next month's dose." c. Proceed with the injection but explain to the patient that other medications are available that may help reduce the unpleasant side effects. d. Notify other staff members to report to the room for a show of force and proceed with the injection, using restraint if necessary.

a. Stop the medication administration procedure and say to the patient, "Tell me more about the side effects you've been having."

A psychiatric technician says, "Little of what takes place on the behavioral health unit seems to be theory based." A nurse educates the technician by identifying which common use of Sullivan's theory? a. Structure of the therapeutic milieu of most behavioral health units b. Frequent use of restraint and seclusion for behavior modification c. Assessment tools based on age-appropriate versus arrested behaviors d. Use of the nursing process to determine the best sequence for nursing actions

a. Structure of the therapeutic milieu of most behavioral health units

A basic level registered nurse works with patients in a community setting. Which groups should this nurse expect to lead? Select all that apply. a. Symptom management b. Medication education c. Family therapy d. Psychotherapy e. Self-care

a. Symptom management b. Medication education e. Self-care

An experienced nurse says to a new graduate, "When you've practiced as long as I have, you will instantly know how to take care of psychotic patients." What is the new graduate's best analysis of this comment? Select all that apply. a. The experienced nurse may have lost sight of patients' individuality, which may compromise the integrity of practice. b. New research findings must be continually integrated into a nurse's practice to provide the most effective care. c. Experience provides mental health nurses with the tools and skills needed for effective professional practice.d. Experienced psychiatric nurses have learned the best ways to care for psychotic patients through trial and error. e. Effective psychiatric nurses should be continually guided by an intuitive sense of patients' needs.

a. The experienced nurse may have lost sight of patients' individuality, which may compromise the integrity of practice. b. New research findings must be continually integrated into a nurse's practice to provide the most effective care.

Several nurses are concerned that agency policies related to restraint and seclusion are inadequate. Which statement about the relationship of substandard institutional policies and individual nursing practice should guide nursing practice? a. The policies do not absolve an individual nurse of the responsibility to practice according to the professional standards of nursing care. b. Agency policies are the legal standard by which a professional nurse must act and therefore override other standards of care. c. In an institution with substandard policies, the nurse has a responsibility to inform the supervisor and leave the premises. d. Interpretation of policies by the judicial system is rendered on an individual basis and therefore cannot be predicted.

a. The policies do not absolve an individual nurse of the responsibility to practice according to the professional standards of nursing care.

A patient being treated in an alcohol rehabilitation unit reveals to the nurse, "I feel terrible guilt for sexually abusing my 6-year-old child before I was admitted." Based on state and federal law, the best action for the nurse to take is to: a. anonymously report the abuse by telephone to the local child abuse hotline. b. reply, "I'm glad you feel comfortable talking to me about it." c. respect the nurse-patient relationship of confidentiality. d. file a written report on the agency letterhead.

a. anonymously report the abuse by telephone to the local child abuse hotline.

A 4-year-old child grabs toys from siblings, saying, "I want that toy now!" The siblings cry, and the child's parent becomes upset with the behavior. Using the Freudian theory, a nurse can interpret the child's behavior as a product of impulses originating in the: a. id. b. ego. c. superego. d. preconscious.

a. id.

An advanced practice nurse determines a group of patients would benefit from therapy in which peers and interdisciplinary staff all have a voice in determining the level of the patients' privileges. The nurse would arrange for: a. milieu therapy b. cognitive therapy c. short-term dynamic therapy d. systematic desensitization

a. milieu therapy

A nurse uses Maslow's hierarchy of needs to plan care for a psychotic patient. Which problem will receive priority? The patient: a. refuses to eat or bathe. b. reports feelings of alienation from family. c. is reluctant to participate in unit social activities. d. needs to be taught about medication action and side effects.

a. refuses to eat or bathe.

A 40-year-old adult living with parents states, "I'm happy but I don't socialize much. My work is routine. When new things come up, my boss explains them a few times to make sure I understand. At home, my parents make decisions for me, and I go along with them." A nurse should identify interventions to improve this patient's: a. self-concept. b. overall happiness. c. appraisal of reality. d. control over behavior.

a. self-concept.

Which patient statements identify qualities of nursing practice with high therapeutic value? (Select all that apply.) "My nurse: a. talks in language I can understand." b. helps me keep track of my medications." c. is willing to go to social activities with me." d. lets me do whatever I choose without interfering." e. looks at me as a whole person with different needs."

a. talks in language I can understand." b. helps me keep track of my medications." e. looks at me as a whole person with different needs."

A patient diagnosed with mental illness asks a psychiatric technician, "What's the matter with me?" The technician replies, "Your wing nuts need tightening." The nurse who overheard the exchange should take action based on: a. violation of the patient's right to be treated with dignity and respect. b. the nurse's obligation to report caregiver negligence. c. preventing defamation of the patient's character. d. supervisory liability.

a. violation of the patient's right to be treated with dignity and respect.

A voluntarily hospitalized patient tells the nurse, "Get me the forms for discharge against medical advice so I can leave now." What is the nurse's best initial response? a. "I can't give you those forms without your health care provider's knowledge." b. "I will get them for you, but let's talk about your decision to leave treatment." c. "Since you signed your consent for treatment, you may leave if you desire." d. "I'll get the forms for you right now and bring them to your room."

b. "I will get them for you, but let's talk about your decision to leave treatment."

Which patient statement would lead a nurse to suspect that the developmental task of infancy was not successfully completed? a. "I have very warm and close friendships." b. "I'm afraid to let anyone really get to know me." c. "I am always right and confident about my decisions." d. "I'm ashamed that I didn't do it correctly in the first place."

b. "I'm afraid to let anyone really get to know me."

A nurse's neighbor asks, "Why aren't people with mental illness kept in state institutions anymore?" What is the nurse's best response? a. "Many people are still in psychiatric institutions. Inpatient care is needed because many people who are mentally ill are violent." b. "Less restrictive settings are now available to care for individuals with mental illness." c. "Our nation has fewer persons with mental illness; therefore fewer hospital beds are needed." d. "Psychiatric institutions are no longer popular as a consequence of negative stories in the press."

b. "Less restrictive settings are now available to care for individuals with mental illness."

Two nursing students discuss career plans after graduation. One student wants to enter psychiatric nursing. The other student asks, "Why would you want to be a psychiatric nurse? All they do is talk. You will lose your skills." Select the best response by the student interested in psychiatric nursing. a. "Psychiatric nurses practice in safer environments than other specialties. Nurse-to-patient ratios must be better because of the nature of patients' problems." b. "Psychiatric nurses use complex communication skills, as well as critical thinking, to solve multidimensional problems. I'm challenged by those situations." c. "I think I will be good in the mental health field. I do not like clinical rotations in school, so I do not want to continue them after I graduate." d. "Psychiatric nurses do not have to deal with as much pain and suffering as medical surgical nurses. That appeals to me."

b. "Psychiatric nurses use complex communication skills, as well as critical thinking, to solve multidimensional problems. I'm challenged by those situations."

Which statement made by the nurse demonstrates the best understanding of nonverbal communication? a. "The patient's verbal and nonverbal communication is often different." b. "When my patient responds to my question, I check for congruence between verbal and nonverbal communication to help validate the response." c. "If a patient is slumped in the chair, I can be sure he's angry or depressed." d. "It's easier to understand verbal communication that nonverbal communication."

b. "When my patient responds to my question, I check for congruence between verbal and nonverbal communication to help validate the response."

You have been working closely with a patient for the past month. Today he tells you he is looking forward to meeting with his new psychiatrist but frowns and avoids eye contact while reporting this to you. Which of the following responses would most likely be therapeutic? a. "A new psychiatrist is a chance to start fresh; I'm sure it will go well for you." b. "You say you look forward to the meeting, but you appear anxious or unhappy." c. "I notice that you frowned and avoided eye contact just now. Don't you feel well?" d. "I get the impression you don't really want to see your psychiatrist—can you tell me why?"

b. "You say you look forward to the meeting, but you appear anxious or unhappy."

Which nursing intervention demonstrates false imprisonment? a. A confused and combative patient says, "I'm getting out of here and no one can stop me." The nurse restrains this patient without a health care provider's order and then promptly obtains an order. b. A patient has been irritating, seeking the attention of nurses most of the day. Now a nurse escorts the patient down the hall, saying, "Stay in your room or you'll be put in seclusion." c. An involuntarily hospitalized patient with suicidal ideation runs out of the psychiatric unit. A nurse rushes after the patient and convinces the patient to return to the unit. d. An involuntarily hospitalized patient with suicidal ideation attempts to leave the unit. A nurse calls the security team and uses established protocols to prevent the patient from leaving.

b. A patient has been irritating, seeking the attention of nurses most of the day. Now a nurse escorts the patient down the hall, saying, "Stay in your room or you'll be put in seclusion."

A patient shows the nurse an article from the Internet about a health problem. Which characteristic of the web site's address most alerts the nurse that the site may have biased and prejudiced information? a. Address ends in ".org." b. Address ends in ".com." c. Address ends in ".gov." d. Address ends in ".net."

b. Address ends in ".com."

A 26-month-old child displays negative behavior, refuses toilet training, and often shouts, "No!" when given directions. Using Freud's stages of psychosexual development, a nurse would assess the child's behavior is based on which stage? a. Oral b. Anal c. Phallic d. Genital

b. Anal

A patient in the emergency department reports, "I hear voices saying someone is stalking me. They want to kill me because I found the cure for cancer. I will stab anyone that threatens me." Which aspects of mental health have the greatest immediate concern to a nurse? Select all that apply. a. Happiness b. Appraisal of reality c. Control over behavior d. Effectiveness in work e. Healthy self-concept

b. Appraisal of reality c. Control over behavior e. Healthy self-concept

Which organization actively seeks to reduce the stigma associated with mental illness through public presentations such as "In Our Own Voice" (IOOV)? a. American Psychiatric Association (APA) b. National Alliance on Mental Illness (NAMI) c. United States Department of Health and Human Services (USDHHS) d. North American Nursing Diagnosis Association International (NANDA-I)

b. National Alliance on Mental Illness (NAMI)

Which student behavior is consistent with therapeutic communication? a. Offering your opinion when asked to convey support. b. Summarizing the essence of the patient's comments in your own words. c. Interrupting periods of silence before they become awkward for the patient. d. Telling the patient he did well when you approve of his statements or actions.

b. Summarizing the essence of the patient's comments in your own words.

Which research evidence would most influence a group of nurses to change their practice? a. Expert committee report of recommendations for practice b. Systematic review of randomized controlled trials c. Nonexperimental descriptive study d. Critical pathway

b. Systematic review of randomized controlled trials

A nurse consistently strives to demonstrate caring behaviors during interactions with patients. Which reaction by a patient indicates this nurse is effective? A patient reports feeling: a. distrustful of others. b. connected with others. c. uneasy about the future. d. discouraged with efforts to improve.

b. connected with others.

A patient diagnosed with schizophrenia believes evil spirits are being summoned by a local minister and verbally threatens to bomb a local church. The psychiatrist notifies the minister. The psychiatrist has: a. released information without proper authorization. b. demonstrated the duty to warn and protect. c. violated the patient's confidentiality. d. avoided charges of malpractice.

b. demonstrated the duty to warn and protect.

A patient is admitted to the psychiatric hospital. Which assessment finding best indicates that the patient has a mental illness? The patient: a. describes coping and relaxation strategies used when feeling anxious. b. describes mood as consistently sad, discouraged, and hopeless. c. can perform tasks attempted within the limits of own abilities. d. reports occasional problems with insomnia.

b. describes mood as consistently sad, discouraged, and hopeless.

A nurse uses Peplau's interpersonal therapy while working with an anxious, withdrawn patient. Interventions should focus on: a. changing the patient's perceptions about self. b. improving the patient's interactional skills. c. using medications to relieve anxiety. d. reinforcing specific behaviors.

b. improving the patient's interactional skills.

After leaving work, a staff nurse realizes that documentation of the administration of a medication to a patient was omitted. This off-duty nurse telephones the unit and tells the nurse, "Please document the administration of the medication I forgot to do. My password is alpha1." The nurse should: a. fulfill the request. b. refer the matter to the charge nurse to resolve. c. access the record and document the information. d. report the request to the patient's health care provider.

b. refer the matter to the charge nurse to resolve.

Which outcome, focused on recovery, would be expected in the plan of care for a patient living in the community and diagnosed with serious and persistent mental illness? Within 3 months, the patient will: a. deny suicidal ideation. b. report a sense of well-being. c. take medications as prescribed. d. attend clinic appointments on time

b. report a sense of well-being

Which statements most clearly reflect the stigma of mental illness? Select all that apply. a. "Many mental illnesses are hereditary." b. "Mental illness can be evidence of a brain disorder." c. "People claim mental illness so they can get disability checks." d. "If people with mental illness went to church, they would be fine." e. "Mental illness is a result of the breakdown of the American family."

c. "People claim mental illness so they can get disability checks." d. "If people with mental illness went to church, they would be fine." e. "Mental illness is a result of the breakdown of the American family."

The parent of a child diagnosed with schizophrenia tearfully asks a nurse, "What could I have done differently to prevent this illness?" Select the nurse's most caring response. a. "Although schizophrenia is caused by impaired family relationships, try not to feel guilty. No one can predict how a child will respond to parental guidance." b. "Most of the damage is done, but there is still hope. Changing your parenting style can help your child learn to cope more effectively with the environment." c. "Schizophrenia is a biological illness with similarities to diabetes and heart disease. You are not to blame for your child's illness." d. "Most mental illnesses result from genetic inheritance. Your genes are more at fault than your parenting."

c. "Schizophrenia is a biological illness with similarities to diabetes and heart disease. You are not to blame for your child's illness."

James is a 42-year-old patient with schizophrenia. He approaches you as you arrive for day shift and anxiously reports, "Last night, demons came to my room and tried to rape me." Which response would be most therapeutic? a. "There are no such things as demons. What you saw were hallucinations." b. "It is not possible for anyone to enter your room at night. You are safe here." c. "You seem very upset. Please tell me more about what you experienced last night." d. "That must have been very frightening, but we'll check on you at night and you'll be safe."

c. "You seem very upset. Please tell me more about what you experienced last night."

In which situations does a nurse have a duty to intervene and report? Select all that apply. a. A peer is unable to write behavioral outcomes. b. A health care provider consults the Physicians' Desk Reference. c. A peer tries to provide patient care in an alcohol-impaired state. d. A team member has violated the boundaries of a vulnerable patient. e. A patient refuses a medication prescribed by a licensed health care provider.

c. A peer tries to provide patient care in an alcohol-impaired state. d. A team member has violated the boundaries of a vulnerable patient.

A patient comments, "I never know the right answer" and "My opinion is not important." Using Erikson's theory, which psychosocial crisis did the patient have difficulty resolving? a. Initiative versus Guilt b. Trust versus Mistrust c. Autonomy versus Shame and Doubt d. Generativity versus Self-Absorption

c. Autonomy versus Shame and Doubt

Operant conditioning will be used to encourage speech in a child who is nearly mute. Which technique would a nurse include in the treatment plan? a. Ignore the child for using silence. b. Have the child observe others talking. c. Give the child a small treat for speaking. d. Teach the child relaxation techniques, then coax speech.

c. Give the child a small treat for speaking.

A nurse volunteers for a committee that must revise the hospital policies and procedures for suicide precautions. Which resources would provide the best guidance? Select all that apply. a. Diagnostic and Statistical Manual of Mental Disorders (fifth edition) (DSM-5) b. State's nurse practice act c. State and federal regulations that govern hospitals d. Summary of common practices of several local hospitals e. American Nurses Association Scope and Standards of Practice for Psychiatric-Mental Health Nursing

c. State and federal regulations that govern hospitals e. American Nurses Association Scope and Standards of Practice for Psychiatric-Mental Health Nursing

A person received an invitation to be in the wedding of a friend who lives across the country. The individual is afraid of flying. What type of therapy should the nurse recommend? a. Psychoanalysis b. Milieu therapy c. Systematic desensitization d. Short-term dynamic therapy

c. Systematic desensitization

A nurse must assess several new patients at a community mental health center. Conclusions concerning current functioning should be made on the basis of: a. the degree of conformity of the individual to society's norms. b. the degree to which an individual is logical and rational. c. a continuum from mentally healthy to unhealthy. d. the rate of intellectual and emotional growth.

c. a continuum from mentally healthy to unhealthy.

A cognitive strategy a nurse could use to assist a very dependent patient would be to help the patient: a. reveal dream content. b. take prescribed medications. c. examine thoughts about being autonomous. d. role model ways to ask for help from others.

c. examine thoughts about being autonomous.

The goal for a patient is to increase resiliency. Which outcome should a nurse add to the plan of care? Within 3 days, the patient will: a. describe feelings associated with loss and stress. b. meet own needs without considering the rights of others. c. identify healthy coping behaviors in response to stressful events. d. allow others to assume responsibility for major areas of own life.

c. identify healthy coping behaviors in response to stressful events.

The parent of a 4-year-old rewards and praises the child for helping a younger sibling, being polite, and using good manners. A nurse supports the use of praise because, according to the Freudian theory, these qualities will likely be internalized and become part of the child's: a. id. b. ego. c. superego. d. preconscious.

c. superego.

A patient should be considered for involuntary commitment for psychiatric care when he or she: a. is noncompliant with the treatment regimen. b. sells and distributes illegal drugs. c. threatens to harm self and others. d. fraudulently files for bankruptcy.

c. threatens to harm self and others.

Which individual with a mental illness may need emergency or involuntary hospitalization for mental illness? The individual who: a. resumes using heroin while still taking methadone. b. reports hearing angels playing harps during thunderstorms. c. throws a heavy plate at a waiter at the direction of command hallucinations. d. does not show up for an outpatient appointment with the mental health nurse.

c. throws a heavy plate at a waiter at the direction of command hallucinations.

Two hospitalized patients fight when they are in the same room. During a team meeting, a nurse asserts that safety is of paramount importance and therefore the treatment plans should call for both patients to be secluded to prevent them from injuring each other. This assertion: a. reveals that the nurse values the principle of justice. b. reinforces the autonomy of the two patients. c. violates the civil rights of the two patients. d. represents the intentional tort of battery.

c. violates the civil rights of the two patients.

The spouse of a patient diagnosed with schizophrenia says, "I don't understand why childhood experiences have anything to do with this disabling illness." Select the nurse's response that will best help the spouse understand this condition. a. "Psychological stress is actually at the root of most mental disorders." b. "We now know that all mental illnesses are the result of genetic factors." c. "It must be frustrating for you that your spouse is sick so much of the time." d. "Although this disorder more likely has a biological rather than psychological origin, the support and involvement of caregivers is very important."

d. "Although this disorder more likely has a biological rather than psychological origin, the support and involvement of caregivers is very important."

A participant at a community education conference asks, "What is the most prevalent type of mental disorder in the United States?" Select the nurse's best response. a. "Why do you ask?" b. "Schizophrenia" c. "Affective disorders" d. "Anxiety disorders"

d. "Anxiety disorders"

An adolescent hospitalized after a violent physical outburst tells the nurse, "I'm going to kill my father, but you can't tell anyone." Select the nurse's best response. a. "You're right. Federal law requires me to keep that information private." b. "Those kinds of thoughts will make your hospitalization longer." c. "You really should share this thought with your psychiatrist." d. "I am required to share information with the treatment team."

d. "I am required to share information with the treatment team."

A nurse assesses a newly admitted patient diagnosed with major depressive disorder. Which statement is an example of "attending"? a. "We all have stress in life. Being in a psychiatric hospital isn't the end of the world." b. "Tell me why you felt you had to be hospitalized to receive treatment for your depression." c. "You will feel better after we get some antidepressant medication started for you." d. "I'd like to sit with you a while so you may feel more comfortable talking with me."

d. "I'd like to sit with you a while so you may feel more comfortable talking with me."

A patient tells a nurse, "I have psychiatric problems and am in and out of hospitals all the time. Not one of my friends or relatives has these problems." Select the nurse's best response. a. "Comparing yourself with others has no real advantages." b. "Why do you blame yourself for having a psychiatric illness?" c. "Mental illness affects 50% of the adult population in any given year." d. "It sounds like you are concerned that others don't experience the same challenges as you."

d. "It sounds like you are concerned that others don't experience the same challenges as you."

An informal group of patients discuss their perceptions of nursing care. Which comment best indicates a patient's perception that his or her nurse is caring? a. "My nurse always asks me which type of juice I want to help me swallow my medication." b. "My nurse explained my treatment plan to me and asked for my ideas about how to make it better." c. "My nurse told me that if I take all the medicines the doctor prescribes I will get discharged soon." d. "My nurse spends time listening to me talk about my problems. That helps me feel like I'm not alone."

d. "My nurse spends time listening to me talk about my problems. That helps me feel like I'm not alone."

In the shift-change report, an off-going nurse criticizes a patient who wears heavy makeup. Which comment by the nurse who receives the report best demonstrates advocacy? a. "This is a psychiatric hospital. Craziness is what we are all about." b. "Let's all show acceptance of this patient by wearing lots of makeup too." c. "Your comments are inconsiderate and inappropriate. Keep the report objective." d. "Our patients need our help to learn behaviors that will help them get along in society."

d. "Our patients need our help to learn behaviors that will help them get along in society."

Cognitive therapy was provided for a patient who frequently said, "I'm stupid." Which statement by the patient indicates the therapy was effective? a. "I'm disappointed in my lack of ability." b. "I always fail when I try new things." c. "Things always go wrong for me." d. "Sometimes I do stupid things."

d. "Sometimes I do stupid things."

A critical care nurse asks a psychiatric nurse about the difference between a diagnosis in the Diagnostic and Statistical Manual of Mental Disorders (DSM-5) and a nursing diagnosis. Select the psychiatric nurse's best response. a. "No functional difference exists between the two diagnoses. Both serve to identify a human deviance." b. "The DSM-5 diagnosis disregards culture, whereas the nursing diagnosis includes cultural variables." c. "The DSM-5 diagnosis profiles present distress or disability, whereas a nursing diagnosis considers past and present responses to actual mental health problems." d. "The DSM-5 diagnosis influences the medical treatment; the nursing diagnosis offers a framework to identify interventions for problems a patient has or may experience."

d. "The DSM-5 diagnosis influences the medical treatment; the nursing diagnosis offers a framework to identify interventions for problems a patient has or may experience."

Which nursing statement is an example of reflection? a. "I think this feeling will pass." b. "So you are saying that life has no meaning." c. "I'm not sure I understand what you mean." d. "You look sad."

d. "You look sad."

Which scenario is an example of a tort? a. The primary nurse does not complete the plan of care for a patient within 24 hours of the patient's admission. b. An advanced practice nurse recommends that a patient who is dangerous to self and others be voluntarily hospitalized. c. A patient's admission status is changed from involuntary to voluntary after the patient's hallucinations subside. d. A nurse gives an as-needed dose of an antipsychotic drug to a patient to prevent violence because a unit is short staffed.

d. A nurse gives an as-needed dose of an antipsychotic drug to a patient to prevent violence because a unit is short staffed.

Therapeutic communication is the foundation of a patient- centered interview. Which of the following techniques is not considered therapeutic? a. Restating b. Encouraging description of perception c. Summarizing d. Asking "why" questions

d. Asking "why" questions

A 26-month-old child displays negative behavior, refuses toilet training, and often shouts, "No!" when given direction. The nurse's counseling with the parent should be based on the premise that the child is engaged in which of Erikson's psychosocial crises? a. Trust versus Mistrust b. Initiative versus Guilt c. Industry versus Inferiority d. Autonomy versus Shame and Doubt

d. Autonomy versus Shame and Doubt

A nurse at a behavioral health clinic sees an unfamiliar psychiatric diagnosis on a patient's insurance form. Which resource should the nurse consult to discern the criteria used to establish this diagnosis? a. A psychiatric nursing textbook b. NANDA International (NANDA-I ) c. A behavioral health reference manual d. Diagnostic and Statistical Manual of Mental Disorders (DSM-5)

d. Diagnostic and Statistical Manual of Mental Disorders (DSM-5)

A nurse wants to find a description of diagnostic criteria for a person diagnosed with schizophrenia. Which resource should the nurse consult? a. U.S. Department of Health and Human Services b. Journal of the American Psychiatric Association c. North American Nursing Diagnosis Association International (NANDA-I) d. Diagnostic and Statistical Manual of Mental Disorders (DSM-5)

d. Diagnostic and Statistical Manual of Mental Disorders (DSM-5)

A patient's history shows intense and unstable relationships with others. The patient initially idealizes an individual and then devalues the person when the patient's needs are not met. Which aspect of mental health is a problem? a. Effectiveness in work b. Communication skills c. Productive activities d. Fulfilling relationships

d. Fulfilling relationships

A nurse listens to a group of recent retirees. One says, "I volunteer with Meals on Wheels, coach teen sports, and do church visitation." Another laughs and says, "I'm too busy taking care of myself to volunteer. I don't have time to help others." These comments contrast which developmental tasks? a. Trust versus Mistrust b. Industry versus Inferiority c. Intimacy versus Isolation d. Generativity versus Self-Absorption

d. Generativity versus Self-Absorption

A person tells a nurse, "I was the only survivor in a small plane crash, but three business associates died. I got anxious and depressed and saw a counselor three times a week for a month. We talked about my feelings related to being a survivor, and now I'm fine, back to my old self." Which type of therapy was used? a. Milieu therapy b. Psychoanalysis c. Behavior modification d. Interpersonal therapy

d. Interpersonal therapy

In a team meeting a nurse says, "I'm concerned whether we are behaving ethically by using restraint to prevent one patient from self-mutilation while the care plan for another patient who has also self-mutilated calls for one-on-one supervision." Which ethical principle most clearly applies to this situation? a. Beneficence b. Autonomy c. Fidelity d. Justice

d. Justice

An adult expresses the wish to be taken care of and often behaves in a helpless fashion. This adult has needs related to which of Freud's stages of psychosexual development? a. Latency b. Phallic c. Anal d. Oral

d. Oral

A newly admitted patient who is acutely psychotic is a private patient of the senior psychiatrist. To whom does the psychiatric nurse who is assigned to this patient owe the duty of care? a. Health care provider b. Profession c. Hospital d. Patient

d. Patient

In the majority culture of the United States, which individual is at greatest risk to be incorrectly labeled mentally ill? a. Person who is usually pessimistic but strives to meet personal goals b. Wealthy person who gives $20 bills to needy individuals in the community c. Person with an optimistic viewpoint about life and getting his or her own needs met d. Person who attends a charismatic church and describes hearing God's voice

d. Person who attends a charismatic church and describes hearing God's voice

A patient underwent psychotherapy weekly for 3 years. The therapist used free association, dream analysis, and facilitated transference to help the patient understand unconscious processes and foster personality changes. Which type of therapy was used? a. Short-term dynamic psychotherapy b. Transactional analysis c. Cognitive therapy d. Psychoanalysis

d. Psychoanalysis

A nurse supports parental praise of a child who is behaving in a helpful way. When the individual behaves with politeness and helpfulness in adulthood, which feeling will most likely result? a. Guilt b. Anxiety c. Loneliness d. Self-esteem

d. Self-esteem

Which belief by a nurse supports the highest degree of patient advocacy during a multidisciplinary patient care planning session? a. All mental illnesses are culturally determined. b. Schizophrenia and bipolar disorder are cross-cultural disorders. c. Symptoms of mental disorders are constant from culture to culture. d. Some symptoms of mental disorders may reflect a person's cultural patterns.

d. Some symptoms of mental disorders may reflect a person's cultural patterns.

Which documentation of a patient's behavior best demonstrates a nurse's observations? a. Isolates self from others. Frequently fell asleep during group. Vital signs stable. b. Calmer and more cooperative. Participated actively in group. No evidence of psychotic thinking. c. Appeared to hallucinate. Patient frequently increased volume on television, causing conflict with others d. Wears four layers of clothing. States, "I need protection from dangerous bacteria trying to penetrate my skin."

d. Wears four layers of clothing. States, "I need protection from dangerous bacteria trying to penetrate my skin."

A psychiatric nurse best implements the ethical principle of autonomy when he or she: a. intervenes when a self-mutilating patient attempts to harm self. b. stays with a patient who is demonstrating a high level of anxiety. c. suggests that two patients who are fighting be restricted to the unit. d. explores alternative solutions with a patient, who then makes a choice.

d. explores alternative solutions with a patient, who then makes a choice.

A nurse psychotherapist works with an anxious, dependent patient. The therapeutic strategy most consistent with the framework of psychoanalytic psychotherapy is: a. emphasizing medication compliance. b. identifying the patient's strengths and assets. c. offering psychoeducational materials and groups. d. focusing on feelings developed by the patient toward the nurse.

d. focusing on feelings developed by the patient toward the nurse.

A single parent is experiencing feelings of inadequacy related to work and family since one teenaged child ran away several weeks ago. The parent seeks the help of a therapist specializing in cognitive therapy. The psychotherapist who uses cognitive therapy will treat the patient by: a. discussing ego states. b. focusing on unconscious mental processes. c. negatively reinforcing an undesirable behavior. d. helping the patient identify and change faulty thinking.

d. helping the patient identify and change faulty thinking.

A nurse cares for an older adult patient admitted for treatment of depression. The health care provider prescribes an antidepressant medication, but the dose is more than the usual adult dose. The nurse should: a. implement the order. b. consult a drug reference. c. give the usual geriatric dosage. d. hold the medication and consult the health care provider.

d. hold the medication and consult the health care provider.

The spouse of a patient who has delusions asks the nurse, "Are there any circumstances under which the treatment team is justified in violating the patient's right to confidentiality?" The nurse must reply that confidentiality may be breached: a. under no circumstances. b. at the discretion of the psychiatrist. c. when questions are asked by law enforcement. d. if the patient threatens the life of another person.

d. if the patient threatens the life of another person.

An example of a breach of a patient's right to privacy occurs when a nurse: a. asks a family to share information about a patient's prehospitalization behavior. b. discusses the patient's history with other staff members during care planning. c. documents the patient's daily behaviors during hospitalization. d. releases information to the patient's employer without consent.

d. releases information to the patient's employer without consent.

An 86-year-old, previously healthy and independent, falls after an episode of vertigo. Which behavior by this patient best demonstrates resilience? The patient: a. says, "I knew this would happen eventually." b. stops attending her weekly water aerobics class. c. refuses to use a walker and says, "I don't need that silly thing." d. says, "Maybe some physical therapy will help me with my balance."

d. says, "Maybe some physical therapy will help me with my balance."


Conjuntos de estudio relacionados

D075 Information Technology (Learning Checks and Quizes)

View Set

Baylor University QBA 2302 Exam 1

View Set

17- Health Insurance Policy Provisons

View Set

Ch 16 Outcome Identification and Planning

View Set

Computer Forensics Final Exam Study Guide

View Set

Cloud Application Security Domain A

View Set